Jump to content
Urch Forums

lsat june 1996 - section 1, question 5


cornofstarchy

Recommended Posts

Hi, I need some help with this one. Here's the question:

 

ADMIN EDIT: QUESTION DELETED. PLEASE DO NOT POST LSAC LSAT QUESTIONS.

 

I've tried it and I keep getting A, B, C, D are true but E is false. The answer key says that D is the correct answer. Can anyone explain to me why? Or is the question worded incorrectly? Should it say, "Which one of the following must be FALSE?" Cause that would make E true... :doh:

 

Oh yea, here are my "solutions"...

 

A)

G,H,J,Q,R,F

H,G,J,Q,R,F

 

B)

G,H,J,Q,R,F

J,G,H,Q,R,F

 

C)

H,G,J,Q,R,F

G,J,H,Q,R,F

 

D)

G,H,J,Q,R,F

G,J,H,Q,R,F

 

E)

Can't because it must end as Q,R,F to fulfill the restriction that R comes immediately before F. Q must always be on day 4.

Link to comment
Share on other sites

Nevermind, I figured it out and here's the answer in case anyone needs it:

 

Solution:

The correct answer is D. To get the answer, simply negate the selection. We know that Q,R,F must be on days 4,5,6 respectively. Choice D says that either H or J is scheduled for day 3. So, if neither J or H is scheduled for day 3, then it should fail.

 

ex) J,H,G,Q,R,F

 

This fails because if G is scheduled for day 3, then Q must be scheduled for day 5. As you can see, Q is not scheduled for day 5. Therefore, D is the correct answer because we cannot put J or H on a day that's not day 3.

Link to comment
Share on other sites

  • 7 months later...

SETUP:

6 days, 1 factory per day, F, G, H, J, Q, R

 

F= 1 or 6

J...Q

QRF

G=3 --> Q=5 therefore Q=/=5 --> G=/=3

 

Choice A.

J G H Q R F works so A might not be true.

 

Choice B.

H G J Q R F works so B might not be true.

 

Choice C.

G H J Q R F works so C might not be true.

 

Choice D.

 

QRF must be last, and G can't be third. So either H or J must be scheduled for day 3.

 

Answer= Choice D

 

I find that I spend significant amounts of time on the choices leading up to the right answer. In the case above, the right answer was easy for me to see. Is there a way that I can usually look ahead and spot the right answer quickly, or is it a waste of time because it is not so easy?

Link to comment
Share on other sites

  • 1 year later...
  • 4 months later...

cornofstarchy-

While your answer is right, I don't think your logic is correct.

 

You said that "Therefore, D is the correct answer because we cannot put J or H on a day that's not day 3."

 

That isn't true. We can't put G on day 3, so day 3 must be J or H.

 

 

We can determine that the sequence is XXXQRF.

 

Then, we recognize that since Q is not in position 5, G cannot be in position 3. So J/H must be in position 3. This is why the answer is D........

 

But if we put J on day 3, we can put H on day 1 or 2, and vice versa.

Link to comment
Share on other sites

Join the conversation

You can post now and register later. If you have an account, sign in now to post with your account.

Guest
Reply to this topic...

×   Pasted as rich text.   Restore formatting

  Only 75 emoji are allowed.

×   Your link has been automatically embedded.   Display as a link instead

×   Your previous content has been restored.   Clear editor

×   You cannot paste images directly. Upload or insert images from URL.

×
×
  • Create New...